LSAT and Law School Admissions Forum

Get expert LSAT preparation and law school admissions advice from PowerScore Test Preparation.

 Administrator
PowerScore Staff
  • PowerScore Staff
  • Posts: 8917
  • Joined: Feb 02, 2011
|
#35268
Complete Question Explanation

Flaw in the Reasoning. The correct answer choice is (A)

Marcia’s conclusion is that “Not all vegetarian diets lead to nutritional deficiencies.” This claim
leaves open the possibility that some vegetarian diets do lead to nutritional deficiencies, but rules out
the possibility that all do. Theodora responds, “You are wrong in claiming that vegetarianism cannot
lead to nutritional deficiencies.” This is clearly a distortion of Marcia’s position and is referred to as
a Straw Man fallacy. The support for both Marcia and Theodora’s claims has no impact on the flaw
in Theodora’s claim and is simply present to provide fodder for incorrect answer choices.

Answer choice (A): This is the correct answer choice. This answer states that Theodora’s reply “is
directed toward disproving a claim that Marcia did not make.” This is a good description of a Straw
Man fallacy and should be noted for reference on future Flaw in the Reasoning questions.

Answer choice (B): Rather than addressing Marcia’s research, Theodora introduces the potential,
indirect effects of vegetarianism. However, this is not necessarily a flaw (even if the effects seem
speculative, at best). In this brief argument, Theodora only has room to advance her own position and
choosing not to address Marcia’s research is a valid argumentative approach.

Answer choice (C): Be careful not to exaggerate Theodora’s position. Theodora suggests that many
meat-based industries will collapse if most people become vegetarians, but this is not the same as
claiming there is no other way for these industries to collapse.

Answer choice (D): While it would be a flaw for Theodora to change the meaning of a term from
Marcia’s argument, Theodora does not use the term differently from Marcia.

Answer choice (E): This answer infers that the inability to afford a nutritionally adequate diet will
lead to vegetarianism, which is not supported by Theodora’s argument.
 netherlands
  • Posts: 136
  • Joined: Apr 17, 2013
|
#8914
So, when I originally read this question I knew something was wrong but didn't know how to vocalize it. But the jist of it seems to be that Theodora attacked a claim that was similar but that Marcia didn't make:

Marcia- Made a claim about individual diets and their effect on a person's ability to gain vitamins and nutrients

Theodora- Disputed this - but did so by talking about vegetarianism as a practice and how its economic effects lead to nutrient problems?

I feel like the difficulty that I'm having understanding a "straw man" fallacy is making it hard for me to see this clearly. But I guess I can see how Theodora misinterpreted or misunderstood Marcia's claim - which would explain why her counter example is seemingly irrelevant.
User avatar
 Dave Killoran
PowerScore Staff
  • PowerScore Staff
  • Posts: 5852
  • Joined: Mar 25, 2011
|
#8926
The Straw Man here is that Marcia says, "not all vegetarian diets lead to nutritional deficiencies," and then Theodora claims that Marcia said, "vegetarianism cannot lead to nutritional deficiencies." Marcia never made that claim, and so when Theodora says that, she recasts Marcia's position. And then she goes off and talks about a somewhat crazy argument (which I think is a device used by the test makers to draw attention away from the flaw that occurs in her first sentence).

Please let me know if that helps. Thanks!
 netherlands
  • Posts: 136
  • Joined: Apr 17, 2013
|
#8929
Ok - so simply put. Theodora argues something completely different than what Marcia did.

And yes, I was severely thrown off by the extra language in both sets. Instead of focusing on the differences in the initial arguments for each person I focused the entire time on the examples that each of them used, the differences in the two of them and trying to determine/vocalize what that difference was.

But I should have been focusing on the argument of each.

I didn't catch the fact that they were both talking about two different things - and instead read it as if they were arguing the same topic but were using reasoning/examples that weren't analogous to one another. I then ended up choosing the right answer because the others were easily eliminated by simple POE (process of elimination.)

Although - still in the back of my mind - I'm not sure if the first sentences alone of each argument are enough for me to have recognized that.

If Marcia and Theodora had both made their initial first two statements, but then Theodora continued on to say something like " Regardless of diet structure, vegetarianism still doesn't provide protein 'A' ". Then she would have used the term "vegetarianism" but is a sense that is analogous and relevant what Marcia is talking about - and so it wouldn't be a misinterpretation, right?

OR.

Maybe I'm completely misunderstanding it and read WAY too far into it, when in fact the issue is a quantitative one - Marcia says "not all" and Theodora says "vegetarianism" ( which basically encompasses "all") - and therefore is refuting a claim that Marcia didn't make (quantitatively speaking). In which case, yea... all the extra verbiage is kind of mumbo-jumbo.

Ugh, if this is the case I'm going to throw my book across the room! :roll:
User avatar
 Dave Killoran
PowerScore Staff
  • PowerScore Staff
  • Posts: 5852
  • Joined: Mar 25, 2011
|
#8936
No, don't do that--you are close! Look again at the first sentence of each..."not all" vs "cannot." Theodora claims that Marcia is saying something she did not actually say, and right there is the main issue, and the one that drives the correct answer choice. Here's an analogy:
  • Marcia: Not all politicians are bad.

    Theodora: You are wrong in claiming that politicians cannot be bad.
That's where the arguments go their separate ways. But, they are talking about similar topics, just from different viewpoints.

Does that make sense?
 netherlands
  • Posts: 136
  • Joined: Apr 17, 2013
|
#8939
Ah, I see. So "not all" vs "cannot". Marcia states that not all cause deficiency. theodora then replies as if Marcia said it cannot cause deficiency at all.

Marcia says - some can, some dont
Theodora responds as if Marcia said- none do.
User avatar
 Dave Killoran
PowerScore Staff
  • PowerScore Staff
  • Posts: 5852
  • Joined: Mar 25, 2011
|
#8942
Pretty much :-D

That's the real issue there, and then Theodora goes off on a wild tangent, which draws the eye away from that original screwup on her part. Tricky, right? Welcome to the LSAT!

As a side note, I suppose we should count ourselves as lucky, because the answer choices are so weak that the correct answer just seems like the obviously best one. They could have made this problem much much harder with better wrong answers.
 lday4
  • Posts: 44
  • Joined: May 05, 2016
|
#23980
For this question, is the key difference that Marcia says "vegetarian diets" and Theodora says "vegetarianism?" If that's the point, then I understand why A is correct in that Marcia is not making a claim about vegetarianism in general. When I first did the question I equated the two and chose B since it seemed like Theodora did answer the claim and just chose to ignore Marcia's premise and give her own.

Thanks!
 Robert Carroll
PowerScore Staff
  • PowerScore Staff
  • Posts: 1787
  • Joined: Dec 06, 2013
|
#24081
Hi lday,

Thanks for the question! Theodora's flaw is that she isn't arguing against Marcia's claim at all. Theodora says "You are wrong in claiming..." and follows that by saying something that Marcia was never claiming in the first place. Marcia claims that not all vegetarian diets will lead to nutritional deficiencies. She then supports that by showing how there is evidence that some such diets will work. Theodora seems to think that Marcia said that vegetarianism won't lead to nutritional deficiencies in any way, whereas Marcia was focusing on the nutritional needs of those adhering to the diets. Theodora's counterargument is beside the point.

Theodora's flaw isn't in ignoring the research, so answer choice (B) is incorrect. Theodora's point makes the research cited by Marcia irrelevant - if Theodora intends instead to argue about the effects of vegetarianism in general, there's no need to counter Marcia's evidence, so it would not be a flaw to ignore that evidence. But the point is that Theodora's argument is about an entirely different issue - thus, the flaw is correctly identified by answer choice (A).

Robert Carroll
 lday4
  • Posts: 44
  • Joined: May 05, 2016
|
#24302
Thanks for the explanation Robert!

Get the most out of your LSAT Prep Plus subscription.

Analyze and track your performance with our Testing and Analytics Package.